Question and Answers Forum

All Questions      Topic List

Integration Questions

Previous in All Question      Next in All Question      

Previous in Integration      Next in Integration      

Question Number 63232 by mathmax by abdo last updated on 01/Jul/19

let B(x,y) =∫_0 ^1 (1−t)^(x−1) t^(y−1)  dt  1) study the convergence of B(x,y)  1) prove that B(x,y)=B(y,x)  prove that B(x,y) =∫_0 ^∞   (t^(x−1) /((1+t)^(x+y) )) dt  2) prove that B(x,y) =((Γ(x).Γ(y))/(Γ(x+y)))  3) prove that Γ(x).Γ(1−x) =(π/(sin(πx)))   for allx ∈]0,1[

$${let}\:{B}\left({x},{y}\right)\:=\int_{\mathrm{0}} ^{\mathrm{1}} \left(\mathrm{1}−{t}\right)^{{x}−\mathrm{1}} {t}^{{y}−\mathrm{1}} \:{dt} \\ $$$$\left.\mathrm{1}\right)\:{study}\:{the}\:{convergence}\:{of}\:{B}\left({x},{y}\right) \\ $$$$\left.\mathrm{1}\right)\:{prove}\:{that}\:{B}\left({x},{y}\right)={B}\left({y},{x}\right) \\ $$$${prove}\:{that}\:{B}\left({x},{y}\right)\:=\int_{\mathrm{0}} ^{\infty} \:\:\frac{{t}^{{x}−\mathrm{1}} }{\left(\mathrm{1}+{t}\right)^{{x}+{y}} }\:{dt} \\ $$$$\left.\mathrm{2}\right)\:{prove}\:{that}\:{B}\left({x},{y}\right)\:=\frac{\Gamma\left({x}\right).\Gamma\left({y}\right)}{\Gamma\left({x}+{y}\right)} \\ $$$$\left.\mathrm{3}\left.\right)\:{prove}\:{that}\:\Gamma\left({x}\right).\Gamma\left(\mathrm{1}−{x}\right)\:=\frac{\pi}{{sin}\left(\pi{x}\right)}\:\:\:{for}\:{allx}\:\in\right]\mathrm{0},\mathrm{1}\left[\right. \\ $$

Commented by mathmax by abdo last updated on 03/Jul/19

1)at V(0)     (1−t)^(x−1) t^(y−1)  ∼t^(y−1)  ⇒ B(x,y)∼∫_0 ^1  (dt/t^(1−y) )  this integral converges  ⇔1−y<1 ⇔y>0  ar V(1)   (1−t)^(x−1) t^(y−1)  ∼ (1−t)^(x−1)     ⇒B(x,y)∼∫_0 ^1    (dx/((1−t)^(1−x) )) wich converges  ⇔1−x<1 ⇔x>0  so  B(x,y) converges ⇔ x>0 and y>0  2)we have B(x,y) =∫_0 ^1 (1−t)^(x−1) t^(y−1) dt =_(1−t=u)    ∫_1 ^o  u^(x−1) (1−u)^(y−1) (−du)  =∫_0 ^1  (1−u)^(y−1) u^(x−1) du =B(y,x)  we say that B(x,y)  is symetric

$$\left.\mathrm{1}\right){at}\:{V}\left(\mathrm{0}\right)\:\:\:\:\:\left(\mathrm{1}−{t}\right)^{{x}−\mathrm{1}} {t}^{{y}−\mathrm{1}} \:\sim{t}^{{y}−\mathrm{1}} \:\Rightarrow\:{B}\left({x},{y}\right)\sim\int_{\mathrm{0}} ^{\mathrm{1}} \:\frac{{dt}}{{t}^{\mathrm{1}−{y}} }\:\:{this}\:{integral}\:{converges} \\ $$$$\Leftrightarrow\mathrm{1}−{y}<\mathrm{1}\:\Leftrightarrow{y}>\mathrm{0} \\ $$$${ar}\:{V}\left(\mathrm{1}\right)\:\:\:\left(\mathrm{1}−{t}\right)^{{x}−\mathrm{1}} {t}^{{y}−\mathrm{1}} \:\sim\:\left(\mathrm{1}−{t}\right)^{{x}−\mathrm{1}} \:\:\:\:\Rightarrow{B}\left({x},{y}\right)\sim\int_{\mathrm{0}} ^{\mathrm{1}} \:\:\:\frac{{dx}}{\left(\mathrm{1}−{t}\right)^{\mathrm{1}−{x}} }\:{wich}\:{converges} \\ $$$$\Leftrightarrow\mathrm{1}−{x}<\mathrm{1}\:\Leftrightarrow{x}>\mathrm{0}\:\:{so}\:\:{B}\left({x},{y}\right)\:{converges}\:\Leftrightarrow\:{x}>\mathrm{0}\:{and}\:{y}>\mathrm{0} \\ $$$$\left.\mathrm{2}\right){we}\:{have}\:{B}\left({x},{y}\right)\:=\int_{\mathrm{0}} ^{\mathrm{1}} \left(\mathrm{1}−{t}\right)^{{x}−\mathrm{1}} {t}^{{y}−\mathrm{1}} {dt}\:=_{\mathrm{1}−{t}={u}} \:\:\:\int_{\mathrm{1}} ^{{o}} \:{u}^{{x}−\mathrm{1}} \left(\mathrm{1}−{u}\right)^{{y}−\mathrm{1}} \left(−{du}\right) \\ $$$$=\int_{\mathrm{0}} ^{\mathrm{1}} \:\left(\mathrm{1}−{u}\right)^{{y}−\mathrm{1}} {u}^{{x}−\mathrm{1}} {du}\:={B}\left({y},{x}\right)\:\:{we}\:{say}\:{that}\:{B}\left({x},{y}\right)\:\:{is}\:{symetric} \\ $$

Commented by mathmax by abdo last updated on 04/Jul/19

we have B(x,y) =∫_0 ^1 (1−t)^(x−1) t^(y−1)  dt   let use the changement t =(1/(u+1)) ⇒  B(x,y) =−∫_0 ^∞    (1−(1/(u+1)))^(x−1) (1/((u+1)^(y−1) )) ((−du)/((u+1)^2 ))  = ∫_0 ^∞    (u^(x−1) /((1+u)^(x−1) )) (1/((1+u)^(y+1) )) du =∫_0 ^∞    (u^(x−1) /((1+u)^(x+y) )) du  so the result is proved .

$${we}\:{have}\:{B}\left({x},{y}\right)\:=\int_{\mathrm{0}} ^{\mathrm{1}} \left(\mathrm{1}−{t}\right)^{{x}−\mathrm{1}} {t}^{{y}−\mathrm{1}} \:{dt}\:\:\:{let}\:{use}\:{the}\:{changement}\:{t}\:=\frac{\mathrm{1}}{{u}+\mathrm{1}}\:\Rightarrow \\ $$$${B}\left({x},{y}\right)\:=−\int_{\mathrm{0}} ^{\infty} \:\:\:\left(\mathrm{1}−\frac{\mathrm{1}}{{u}+\mathrm{1}}\right)^{{x}−\mathrm{1}} \frac{\mathrm{1}}{\left({u}+\mathrm{1}\right)^{{y}−\mathrm{1}} }\:\frac{−{du}}{\left({u}+\mathrm{1}\right)^{\mathrm{2}} } \\ $$$$=\:\int_{\mathrm{0}} ^{\infty} \:\:\:\frac{{u}^{{x}−\mathrm{1}} }{\left(\mathrm{1}+{u}\right)^{{x}−\mathrm{1}} }\:\frac{\mathrm{1}}{\left(\mathrm{1}+{u}\right)^{{y}+\mathrm{1}} }\:{du}\:=\int_{\mathrm{0}} ^{\infty} \:\:\:\frac{{u}^{{x}−\mathrm{1}} }{\left(\mathrm{1}+{u}\right)^{{x}+{y}} }\:{du}\:\:{so}\:{the}\:{result}\:{is}\:{proved}\:. \\ $$

Terms of Service

Privacy Policy

Contact: info@tinkutara.com